4
$\begingroup$

I want to express the square root of NOT as a time-dependent unitary matrix such that each $n$ units of time, the square root of NOT is produced.

More precisely, I want to find a $U(t_0,t_1)$ such that $U(t_0,t_1) = \sqrt{\text{NOT}}$, if $t_1-t_0=n$ for some $n$.

One possible solution is to express $\sqrt{\text{NOT}}$ as a product of rotation matrices, and then, parametrize the angles in a clever way to depend on the time. But I do not know how to express $\sqrt{\text{NOT}}$ as a product of rotation matrices.

Any help?

$\endgroup$
1
  • 2
    $\begingroup$ Use $U(t)=\exp[-it\sigma_x]$. $\endgroup$ Jul 4, 2018 at 18:03

1 Answer 1

4
$\begingroup$

$$ \sqrt{NOT} = e^{(\frac{i \pi}{4} I_2 - \frac{i \pi}{4} \sigma_x)}\\ U(t) = e^{\frac{t-t_0}{t_1 - t_0} (\frac{i \pi}{4} I_2 - \frac{i \pi}{4} \sigma_x)} $$

$\endgroup$
5
  • $\begingroup$ I think the $U(t)$ you proposed is not unitary (so not valid). Am I right? $\endgroup$ Jul 5, 2018 at 18:33
  • $\begingroup$ If you write that $U(t)=e^{i (t-t_0) H}$, $H$ would be $\frac{\pi}{4(t_1-t_0)} (I_2 - \sigma_x)$. Check that $H$ is Hermitian. $\endgroup$
    – AHusain
    Jul 5, 2018 at 18:43
  • $\begingroup$ Sorry for all the questions: Doesn't have to be skew-Hermitian to ensure unitarity? That is, if $A^\dagger=-A$ then $e^A$ is unitary. Here the matrix $\frac{t-t_0}{t_1-t_0}\frac{i\pi}4(I_2-\sigma_x)$ seems to be Hermitian, which just ensure that $U(t)$ is Hermitian, but not unitary. $\endgroup$ Jul 5, 2018 at 22:28
  • 1
    $\begingroup$ $(i H)^\dagger = -i H^\dagger = - i H$ $\endgroup$
    – AHusain
    Jul 5, 2018 at 22:33
  • $\begingroup$ Great. Thanks. Then I am doing something wrong. Take $\delta_t\frac{i\pi}4(I_2-\sigma_x)$ and diagonalize it, getting $SDS^{-1}$, then $U(t)=Se^DS^{-1}$, and so you can transform the expression of $U(t)$ into $\frac 12\begin{bmatrix} 1+i^{\delta_t} & 1-i^{\delta_t} \\ 1-i^{\delta_t} & 1+i^{\delta_t}\end{bmatrix}$, but that matrix is not unitary. So I do not understand where I am making a mistake. $\endgroup$ Jul 5, 2018 at 22:36

Your Answer

By clicking “Post Your Answer”, you agree to our terms of service and acknowledge you have read our privacy policy.

Not the answer you're looking for? Browse other questions tagged or ask your own question.